Neuro Test

¡Supera tus tareas y exámenes ahora con Quizwiz!

A 56 year old female client asks why the nurse is assessing her for a stroke. Which manifestations did the nurse use to make this assessment decision? (Select all that apply.) a.Nausea b.Hiccups c.Itchy skin d.Chest pain e.Palpitations

A 56 year old female client asks why the nurse is assessing her for a stroke. Which manifestations did the nurse use to make this assessment decision? (Select all that apply.) a.Nausea b.Hiccups c.Itchy skin d.Chest pain e.Palpitations

A patient with a brain injury is not able to respond appropriately to sensory stimulation. What should the nurse do to ensure that this patient does not develop skin breakdown? (Select all that apply.) a. Protect bony prominences b. Assess the skin every 2 hours c. Moisturize the skin as needed d. Apply paper tape over wounds e. Turn and reposition every 2 hours

ANS: A, B, C, E To protect the skin of a patient with an alteration in sensation, the nurse should protect bony prominences, assess the skin every 2 hours, moisturize the skin as needed, and turn and reposition the patient every 2 hours. D. Paper tape can adhere to delicate skin tissue and cause tears.

A 22-year-old female patient recovering from a craniotomy begins crying and asking for her mother who is sleeping in the visitors lounge. The patients Glasgow Coma Scale (GCS) of 15 and pupils are equal and reactive. What nursing action would be most appropriate at this time? a. Ask the mother to come and stay with the patient. b. Administer an as-needed sedative to calm the patient. c. Notify the neurosurgeon that the patient is upset and crying. d. Reassure the patient, and sit with her until she falls back asleep.

ANS: A A GCS of 15 and other assessment findings show the patient is stable. She has been through tremendous stress and wants her mother. D. There is no reason not to ask the mother to come and stay with her. B. A sedative is not indicated and may mask assessment findings. C. There is no reason to contact the surgeon.

The nurse is providing care for a client admitted to the hospital after a motor vehicle accident. After being informed by family members that the patient is deaf and mute, which action should the nurse take? a. Avoid use of the Glasgow Coma Scale. b. Consider the Babinski response invalid. c. Utilize a three-point scale to grade muscle strength. d. Perform the Romberg test with the patient in a seated position.

ANS: A A limitation of the Glasgow Coma scale is that if one of the three components cannot be measured, the resulting score is of no use. Because it is not possible to fully evaluate verbal responses for this patient, the scale should not be used. B. The patients inability to hear or speak will not affect the results of a Babinski reflex assessment. C. The patients inability to hear or speak will not affect the scale used to grade muscle strength. D. The Romberg test is not completed in the seated position.

The nurse is preparing material about the neurological system as part of major presentation. What should the nurse include that explains the purpose of white matter? a. Carries either sensory or motor impulses b. Location of white blood cells within the brain c. Protects the spinal nerves from potential injury d. Regulates movement and responses to external stimuli

ANS: A A nerve tract is a group of thickly myelinated neurons within the central nervous system; such tracts within white matter appear white due to the myelin sheaths. A nerve tract within the spinal cord carries either sensory or motor impulses. B. The white matter does not store white blood cells within the brain. C. The vertebral column protects the spinal nerves from potential injury. D. The white matter does not regulate movement and responses to external stimuli.

The nurse is providing care for an 87-year-old woman who is recovering from a cerebral vascular accident. Which precaution should the nurse take after noting the patient has a positive Romberg test? a. Institute fall-risk precautions. b. Provide small, frequent meals. c. Request a footboard and splints. d. Darken the room and reduce stimuli.

ANS: A A positive Romberg test in an older adult is expected as a result of normal aging changes in the cerebellum. Be sure to protect the patient with a positive result from falls. A gait belt may be helpful when assisting the patient with ambulation. B. C. D. The patient does not need a change in meal frequency, a footboard, splints, or a darkened room to reduce stimuli.

The nurse suspects that a patient will be diagnosed as being in a comatose state based upon the Glasgow Coma Scale score. What score does the patient need to have to be identified as comatose? a. 7 b. 9 c. 11 d. 13

ANS: A A score of less than 7 indicates a comatose patient, and a score of 15 indicates the patient is fully alert and oriented. B. C. D. These scores indicate that the patient has some neurological dysfunction but is not considered comatose.

The nurse is planning care for a patient with advancing Alzheimers disease. Which nursing diagnosis should be the priority for this patient? a. Risk for Injury b. Noncompliance c. Bathing Self-Care Deficit d. Ineffective Role Performance

ANS: A According to Maslows hierarchy, safety needs come before higher-level needs. In addition, injury is the most life- and health-threatening problem. B. C. D. These nursing diagnoses can be addressed after the Risk for Injury has been addressed.

The nurse is assessing a patients pupils for reactivity to light. Which cranial nerve (CN) is being tested? a. CN III b. CN IV c. CN VI d. CN XII

ANS: A CN III controls movement of the eyeball and constriction of the pupil for bright light or near vision. B. C> CN IV and VI control eyeball movement. D. CN XII controls tongue movement.

A patient recovering from a brain injury is having difficulty completing activities of daily living. What should the nurse suggest to help this patient recover independence with self-care? a. Occupational therapy consultation b. Transfer to a rehabilitation facility c. Hire long-term private care assistance d. Cognitive stimulation to keep on track

ANS: A For the patient having difficulty completing self-care actions, an occupational therapy consultation might be needed. An occupational therapist is trained to assist patients to manage ADLs within health limitations. B. The patient does not need to be transferred to a rehabilitation facility. C. Long-term private care assistance is not needed while the patient is still hospitalized. This might need to be an option once discharged to home. D. Cognitive stimulation is not an option for this type of health problem.

A patient in the plateau stage of Guillain-Barre syndrome is frustrated because there has not been any improvement in manifestations for 5 days. What should the nurse explain to the patient? a. The manifestations can last up to 2 weeks b. The manifestations can last up to 3 weeks c. The manifestations can last up to 6 months d. The manifestations can last up to 24 months

ANS: A GBS is divided into three stages. The second stage is the plateau stage, when symptoms are most severe but progression has stopped. It can last from 2 to 14 days. Patients may become discouraged if no improvement is evident. B. The first stage starts with the onset of symptoms and lasts until the progression of symptoms stops. This stage can last from 24 hours to 3 weeks. C. D. Axonal regeneration and remyelination occur during the third state, recovery. This stage lasts from 6 to 24 months and symptoms slowly improve.

An adolescent sustains an injury while swimming in a river. Friends bring the adolescent to the riverbank and note that the adolescent is conscious and breathing but not moving any extremities. What should the friends do next? a. Immobilize the boy, and call for help. b. Push on his stomach to rid his lungs of water. c. Use a four-man carry to take the boy to safety. d. Turn him onto his stomach to allow water to drain from his lungs.

ANS: A He has a possible spinal cord injury and should be immobilized and not moved to prevent further injury. B. D. Since he is breathing, providing respiratory interventions can wait until emergency personnel are present. C. The patient should not be moved.

The nurse suspects a patient with a spinal cord injury is experiencing spinal shock. What did the nurse assess to come to this conclusion? a. Flaccid paralysis and lack of sensation below the level of the injury b. Loss of voluntary motor control, but presence of reflex activity below the level of the injury c. Falling blood pressure and rising pulse accompanied by reduced level of consciousness d. Loss of motor control below the level of the injury with sensations of touch and position intact

ANS: A Immediately following a spinal cord injury, the cord below the injury stops functioning completely. This leads to a loss of motor and sensory functions as well as reflexes. B. C. D. Falling blood pressure and rising pulse indicate shock from blood loss or cardiovascular cause.

A patient who has had a seizure is crying, saying life is over, and that working and driving will no longer be possible. Which response by the nurse is most appropriate? a. With good seizure control, you should be able to work and drive again. b. Maybe the social worker can help you identify some alternative activities. c. You may be able to work again in time; you can use public transportation. d. You should be able to discontinue your medication within a month and return to work.

ANS: A Patients with poorly controlled seizures should not operate motor vehicles. If seizures can be well controlled with medication, then driving is possible. B. C. These are inappropriate statements until it is known how the patient responds to medication. D. Discontinuing the medication may be done after an extended seizure-free Test Bank - Understanding Medical-Surgical Nursing 6e (Williams and Hopper) 795 period but not after a month.

A patient is prescribed phenytoin (Dilantin) for seizure activity. What should the nurse include when teaching the patient about this medication? a. Be sure to brush and floss your teeth daily. b. Be sure to arrange for regular checkups for potassium levels. c. You may notice some vision changes while taking this drug. d. You may experience shortness of breath as a side effect of the drug.

ANS: A Regular dental care is important because of risk for gingival hyperplasia. B. C. D. Shortness of breath, vision changes, and potassium imbalance are not associated with phenytoin therapy.

A patient with suspected spinal cord and head injuries has a Glasgow Coma Scale score of 15; blood pressure 130/82 mm Hg, pulse 102 beats/min, respirations 20 breaths/min, and temperature 98F (36.6C). What is the most important nursing intervention during the initial care of the patient? a. Avoid moving the patient. b. Check the extremities for range of motion. c. Turn the patient to the side to avoid aspiration. d. Keep the head of the bed elevated 30 degrees.

ANS: A The first priority is prevention of further spinal cord injury, so the patient must be kept flat without movement of the trunk. B. C. D. Checking range of motion, turning, or elevating the head could cause further injury.

The nurse is determining care for a patient with acute migraine headaches. What should the nurse teach the patient to do first in order to determine a plan of care for the headaches? a. Keep a headache diary. b. Avoid sugar and caffeine. c. Avoid bright light and noise. d. Avoid taking analgesics until the cause has been determined.

ANS: A The patient can identify aggravating factors by keeping a headache diary for a time, recording the time of day the headache occurs, foods eaten or other aggravating factors, description of the pain, identification of associated symptoms such as nausea or visual disturbances, and other factors related to headache symptoms. B. C. Avoiding foods or lights and noise might be helpful, but they do not help identify factors causing headaches. D. Avoiding analgesics is unrealistic.

A patient with quadriplegia from a C5 injury is wearing a Halo vest and begins to experience a throbbing headache and nausea. What should the nurse do first? a. Check the patients blood pressure. b. Do a digital rectal examination for the presence of an impaction. c. Notify the charge nurse or physician immediately of the patients headache. d. Advise the patient that sitting in the wheelchair will help relieve the headache.

ANS: A The patient has symptoms of autonomic dysreflexia, which can occur in patients with injuries above the T6 level. B. Once blood pressure is checked and under control, then further assessment can be done to find the source of the problem. D. Sitting will not fix the problem. C. The physician or charge nurse can be notified as needed once further assessment and intervention are completed.

A patient with multiple sclerosis is unable to verbally communicate needs. Which approach should the nurse use to assess this patient for pain? a. Observe for grimacing or agitation b. Ask the patient to press the call light if pain is present c. Schedule pain medication every 4 hours around the clock d. Offer pain medication every shift with routine medications

ANS: A The patient may not be able to verbally state that pain is present. The nurse should assess the patient for nonverbal signs of pain or distress, such as restlessness, agitation, and grimacing. B. The patient might not be able to press the call light. C. D. Pain medication should not be routinely provided around the clock or with other medications without completing an adequate pain assessment.

The vital signs for a client with a possible head injury were on admission: blood pressure 128/72 mm Hg, pulse 90 beats/min, and respirations 66 breaths/min. Which vital sign assessment conducted four hours later most likely indicates the presence of increased intracranial pressure (ICP)? a. Blood pressure 172/68 mm Hg, pulse 42 beats/min, respirations 10 breaths/min b. Blood pressure 160/90 mm Hg, pulse 112 beats/min, respirations 16 breaths/min c. Blood pressure 130/72 mm Hg, pulse 50 beats/min, respirations 24 breaths/min d. Blood pressure 100/70 mm Hg, pulse 120 beats/min, respirations 30 breaths/min

ANS: A Vital sign changes are a late indication of increasing ICP. Cushings response is a classic late sign of increased ICP. Cushings response (or Cushings triad) is characterized by bradycardia, bradypnea, and arterial hypertension (increasing systolic blood pressure while diastolic blood pressure remains the same), resulting in widening pulse pressure. B. These vital signs indicate tachycardia. C. These vital signs indicate tachypnea. C. These vital signs indicate both tachycardia and tachypnea.

The nurse is reviewing the vital sign measurements for a patient with a neurological problem. When analyzing these measurements what should the nurse recall as the part of the brain that regulates heart rate and blood pressure? a. Medulla b. Cerebrum c. Cerebellum d. Hypothalamus

ANS: A Within the medulla are cardiac centers that regulate heart rate, respiratory centers that regulate breathing, and vasomotor centers that regulate the diameter of blood vessels and therefore blood pressure. B. D. The hypothalamus and cerebrum have varied functions. C. The functions of the cerebellum are concerned with the involuntary aspects of voluntary movement: coordination, the appropriate direction and endpoint of movements, and the maintenance of posture and balance or equilibrium.

The nurse is caring for an individual who has a tension headache. Which interventions should be included in the patients plan of care? (Select all that apply.) a. Massage b. Moist heat c. Ergotamine d. Dark glasses e. Aerobic exercise f. Cold compresses

ANS: A, B Symptom management may include the use of relaxation techniques, massage of the affected muscles, rest, localized heat application, nonnarcotic analgesics, and appropriate counseling. F. Heat, not cold, will relax muscles. C. Ergotamine may be helpful for a migraine headache. E. Rest, not exercise, may help. D. Photophobia is not typically associated with a tension headache.

The nurse is caring for a patient with post-polio syndrome. What should the nurse explain to the patient in preparation for discharge? (Select all that apply.) a. Engage in moderate exercise. b. Ensure an adequate amount of rest each day. c. Schedule periods of intense physical activity. d. Spend several hours each day in direct sunlight. e. Reduce the intake of high protein high fat foods.

ANS: A, B There is no specific therapeutic regimen for this disorder. Symptoms seem to be best controlled by rest and moderate exercise without pushing the limits of tolerance. C. Intense physical activity could exacerbate symptoms of the disorder. D. Sunlight is not an identified treatment for post-polio syndrome. E. High protein and high fat foods are not implicated in the development or treatment of this disorder

The nurse is caring for a patient with an acute brain injury. Which interventions should the nurse use to prevent increased intracranial pressure in this patient? (Select all that apply.) a. Avoid hip flexion. b. Administer stool softeners. c. Keep head of bed elevated 30 degrees. d. Encourage deep breathing and coughing. e. Administer opioid analgesics for headache.

ANS: A, B, C Elevation of the head of the bed may help reduce intracranial pressure (ICP). Stool softeners prevent straining, which can increase ICP. Hip flexion may also increase ICP. D. E. Coughing can increase ICP, and opioid analgesics make neurological assessment difficult.

The nurse suspects a patient is experiencing a sympathetic response. What manifestations should the nurse expect the patient to demonstrate this response? (Select all that apply.) a. Relaxation of bladder b. Decrease in peristalsis c. Dilation of bronchioles d. Decrease in heart rate to normal e. Increase in salivary gland secretion

ANS: A, B, C When the sympathetic nervous system is activated, the heart rate increases, vasodilation in skeletal muscles supplies them with more oxygen, the bronchioles dilate to take in more air, and the liver changes glycogen to glucose to provide energy. Relatively less important activities such as digestion (and salivation) are slowed, and vasoconstriction in the skin and viscera permits greater blood flow to more vital organs such as the brain, heart, and muscles. The bladder muscle relaxes, and the sphincter constricts to prevent urination.

After frequent examinations a patient is diagnosed with amyotrophic lateral sclerosis (ALS). Which test results should the nurse review as confirmation of this diagnosis? (Select all that apply.) a. Nerve biopsy b. Electroencephalogram c. Nerve conduction velocity d. Analysis of cerebrospinal fluid e. CT scan of the brain and spinal cord

ANS: A, B, C, D Diagnosis of ALS is made based on clinical symptoms. Additional tests such as nerve biopsy, electroencephalogram, nerve conduction velocity and CSF analysis may be done to rule out other conditions. E. CT scan of the brain and spinal cord are not used to diagnose ALS.

The nurse is caring for a patient diagnosed with bacterial meningitis. Which medications should the nurse expect to be prescribed for this patient? (Select all that apply.) a. Analgesics b. Antibiotics c. Antipyretics d. Anticoagulants e. Anti-inflammatory agents

ANS: A, B, C, E Analgesics are given to lessen head and neck pain. Antibiotics are administered for bacterial meningitis. Antipyretics such as acetaminophen are used to control the fever. Anti-inflammatory agents are given to decrease swelling. D. Anticoagulants are not routinely administered in the treatment of bacterial meningitis.

The nurse is teaching a patient with myasthenia gravis how to recognize a cholinergic crisis. What manifestations should the nurse include in this teaching? (Select all that apply.) a. Diarrhea b. Salivation c. Vomiting d. Difficulty speaking e. Abdominal cramping f. Increased bronchial secretions

ANS: A, B, C, E, F Symptoms of cholinergic crisis can be remembered with the acronym SLUDGE: salivation, lacrimation, urination, diarrhea, gastrointestinal cramping, and emesis. A severe crisis has been described as liquid pouring out of every body orifice. D. Difficulty speaking is not a manifestation of a cholinergic crisis.

The nurse suspects that a patient is experiencing increasing intracranial pressure. What observations did the nurse make to come to this conclusion? (Select all that apply.) a. Headache b. Rising temperature c. Decreasing systolic pressure d. Dilated pupil on affected side e. Decreasing level of consciousness (LOC)

ANS: A, B, D, E Headache, increasing systolic pressure, decreasing LOC, dilated pupil on affected side, and rising temperature are all signs of increased ICP. C. Decreasing systolic blood pressure is not associated with increased intracranial pressure.

The nurse is caring for a patient scheduled for a computed tomography (CT) scan with contrast. What should be included in pre-procedure preparation? (Select all that apply.) a. Check blood urea nitrogen (BUN) and creatinine levels. b. Question the patient about allergies to dye, shellfish, or iodine. c. Determine if the patient has aneurysm clips or metal pins in the body. d. Explain to the patient that a sensation of warmth may be felt when dye is injected. e. Tell the patient to report any nausea, itchiness, or difficulty breathing during the scan.

ANS: A, B, D, E Patients who are receiving dye should be warned that they may feel a sensation of warmth following the injection; warmth in the groin area may make them feel as though they have been incontinent of urine. Nausea, diaphoresis, itching, or difficulty breathing may indicate allergy to the dye and should be reported immediately to the physician or nurse practitioner. Sedation may be required for patients who are agitated or disoriented. Patients who are in pain may require pain medication before the examination. The patient should be questioned about any allergies to contrast material, iodine, or shellfish. The BUN and creatinine levels should be checked before administration of contrast material because it is excreted through the kidneys. Patients with elevated BUN and creatinine or known renal disease may be unable to tolerate the contrast material. C. Clips or metal pins in the body would be assessed if the patient were scheduled for an MRI.

The nurse is planning to use the FOUR tool to assess a patients neurological functioning. In which areas should the nurse collect data when using this tool? (Select all that apply.) a. Reflexes b. Eye response c. Verbal response d. Motor movement e. Breathing pattern

ANS: A, B, D, E The FOUR tool measures data from four categories: eye response, motor movement, reflexes, and breathing pattern. C. A major benefit of using the FOUR is that no evaluation of verbal response is necessary.

A patient with bacterial meningitis has an elevated temperature. Which actions should the nurse take to reduce this patients temperature? (Select all that apply.) a. Use tepid sponge baths as needed b. Monitor temperature every 4 hours c. Apply ice to the groin every 2 hours d. Administer antipyretics as prescribed e. Place on a cooling blanket if available

ANS: A, B, D, E To help reduce hyperthermia associated with bacterial meningitis the nurse should use tepid sponge baths as needed, monitor the temperature every 4 hours, administer antipyretics as prescribed, and place on a cooling blanket if available. C. Ice should not be applied to the groin because shivering could occur.

While observing the neurologist complete a neurological examination the nurse notes that a patient does not have a left patellar reflex. In which areas should the nurse consider the patient has a dysfunction? (Select all that apply.) a. Spinal cord b. Femoral nerve c. Anterior fibula muscle d. Posterior tibial muscle e. Quadriceps femoris muscle

ANS: A, B, E If the patellar reflex is absent, the problem might be in the quadriceps femoris muscle, the femoral nerve, or the spinal cord itself. C. D. The absence of a patellar reflex does not suggest that a problem exists within the anterior fibula or posterior tibial muscles.

A patient is experiencing a new onset of a seizure. What should the nurse include in this patients plan of care? (Select all that apply.) a. Suction if necessary. b. Monitor vital signs when possible. c. Place the patient in a supine position. d. Restrain the patient to prevent injury. e. Observe and document progression of symptoms. f. Protect the patient from injury by removing nearby objects.

ANS: A, B, E, F The patient should be protected from injury. Vital signs are monitored when able, if it will not injure the patient. Observing symptom progression can help diagnose the type of seizure, and suction may be necessary to prevent aspiration. C. The patient should be placed in a side-lying position if possible, not supine, to prevent aspiration. D. Restraining the patient increases the risk for injury.

The nurse is explaining the difference between Bells palsy and trigeminal neuralgia to a nursing assistant. What should the nurse include as characteristics of Bells palsy? (Select all that apply.) a. Drooling b. Facial droop c. Sudden onset d. Airflow sensitivity e. Sensitivity to temperature f. Loss of taste over anterior part of tongue

ANS: A, B, F Patients with Bells palsy have a facial droop, variable symptom onset, changes in taste, and drooling. C. D. E. Patients with trigeminal neuralgia (TN) experience sudden onset and temperature and airflow sensitivity.

The nurse is providing care for a patient with a history of aspiration. Which foods should the nurse remove from patients tray? (Select all that apply.) a. Coffee b. Ice cream c. Fruit juice d. Applesauce e. Ground chicken f. Bread with butter

ANS: A, C Patients with swallowing difficulty (dysphagia) may have better success with foods or thick liquids rather than thin fluids. Thin fluids are more easily aspirated and should be altered (thickened) prior to drinking. B. D. E. F. These foods are less likely to cause aspiration.

The nurse is planning care for a patient with a migraine headache. Which actions should the nurse include in this plan of care? (Select all that apply.) a. Rest b. White noise c. A dark, quiet room d. Sumatriptan (Imitrex) e. Acetaminophen (Tylenol) f. Pseudoephedrine (Sudafed)

ANS: A, C, D A dark room and rest help reduce stimulation during a migraine headache. Sumatriptan is a medication available used for migraine relief. E. F. Acetaminophen and decongestants may be helpful for sinus headaches. B. Stimulation (noise and light) may worsen a migraine.

After collecting data the nurse determines that a patient is experiencing cluster headaches. What information did the nurse use to come to this conclusion? (Select all that apply.) a. Throbbing and excruciating pain Test Bank - Understanding Medical-Surgical Nursing 6e (Williams and Hopper) 792 b. Bright sunlight causes severe eye pain c. Sudden onset at the same time during the night d. Pain that affects one side of the nose, eye and forehead e. The eye on the side of the headache is bloodshot and tearing

ANS: A, C, D, E Manifestations of cluster headaches include throbbing and excruciating pain, sudden onset typically at the same time of night, pain that is be unilateral, affecting the nose, eye, and forehead, and a bloodshot, teary appearance of the affected eye. B. Photophobia is not a manifestation of cluster headaches.

A patient with a spinal cord injury at T3T4 experiences a sudden increase in blood pressure (BP) and has cool, pale, gooseflesh skin on the lower extremities. What should the nurse do while awaiting physician orders? (Select all that apply.) a. Monitor BP every 5 minutes. b. Place the patient in supine position. c. Place elastic stockings on the patients legs. d. Check to see if the indwelling catheter is patent. e. Perform a rectal examination to determine if impaction is present.

ANS: A, D, E The patient is experiencing autonomic dysreflexia, which can cause hypertension and bradycardia. The nurse should monitor BP and then check for catheter patency and impaction, both of which can cause dysreflexia. B. C. The patient should be placed in high Fowlers position, and elastic stockings should be removed to allow blood to pool and reduce BP. PTS: 1 DIF: Moderate

The nurse is caring for a patient who is being tested for possible myasthenia gravis (MG). Which early symptoms of myasthenia gravis should the nurse document in the medical record? (Select all that apply.) a. Ptosis b. Nausea c. Tremor d. Confusion e. Weakness f. Numbness of the extremities

ANS: A, E The hallmark of MG is increased muscle weakness during activity and improvement in muscle strength after rest. Patients often present with drooping of the eyelids (ptosis). F. Numbness and tingling of the extremities is characteristic of multiple sclerosis (MS), not MG. B. C. D. Nausea, tremor and confusion are not characteristics of MG.

The nurse completes data collection on a newly admitted older patient. Which finding is considered abnormal in an aging patient and should be reported to the physician? (Select all that apply.) a. Depression b. Forgetfulness c. Altered sleep patterns d. Decreased postural stability e. Fine motor tremors of the hands f. Decreased problem-solving ability

ANS: A, E With age the brain loses neurons, but this is only a small percentage of the total and is not the usual cause of mental impairment in older adults; far more common causes of mental changes include depression, malnutrition, hypotension, and the side effects of medications. Some forgetfulness is to be expected, however, as is a decreased ability for problem solving, altered sleep patterns, and a decrease in postural stability. Fine motor tremors in the hands are a symptom of Parkinsons disease and are considered an abnormal finding.

The nurse is preparing to conduct a Romberg test with a patient. For how many seconds should the nurse explain to the patient that the position will need to be held? a. 10 b. 20 c. 30 d. 40

ANS: B A negative Romberg test means that the patient experiences minimal swaying for up to 20 seconds. A patient who experiences swaying or who leans to one side is said to have a positive Romberg test. A. The test needs to be conducted for longer than 10 seconds. C. D. The test does not need to be conducted for 30 or 40 seconds.

A patient is diagnosed with a health problem that alters the way impulses are conducted in the neurological system. When reviewing this information with the patient, which part of a neuron should the nurse teach carries impulses toward the cell body? a. Axon b. Dendrite c. Schwann cell d. Myelin sheath

ANS: B A neuron may have one or many dendrites, which are extensions that carry impulses toward the cell body. A. A neuron has one axon that transmits impulses away from the cell body. C. In the peripheral nervous system, axons and dendrites are wrapped in specialized neuroglial cells called Schwann cells. The concentric layers of cell membrane of a Schwann cells plasma membrane form the myelin sheath. D. Myelin is a phospholipid that electrically insulates neurons from one another.

A patient who is severely brain damaged has decerebrate posturing with extended extremities. In which area of the brain should the nurse suspect the patient has sustained damage? a. Cerebrum b. Brain stem c. Cerebellum d. Hypothalamus

ANS: B Abnormal extension posturing, or decerebrate posturing, indicates damage in the area of the brain stem. A. Decorticate posturing indicates significant impairment of cerebral functioning. C. D. There are no specific postures associated with damage to the cerebellum or hypothalamus.

The nurse is providing post-procedure care for a patient recovering from a lumbar puncture. Which order should the nurse anticipate for this patient? a. Keep the patient NPO for 4 hours. b. Have the patient lie flat for 6 hours. c. Monitor the patients pedal pulses every 4 hours. d. Keep the head of the bed elevated 30 degrees for 8 hours.

ANS: B After the lumbar puncture is completed, instruct the patient to remain on bedrest with the head of the bed flat for 6 to 8 hours, as ordered by the physician, and to increase oral intake of fluids. Keeping the head flat decreases the likelihood of leakage of cerebrospinal fluid from the puncture site, which can result in a severe headache. C. Pedal pulses are important for angiograms. A. The patient does not need to be kept at nothing by mouth status for 4 hours. D. Elevating the head of the bed could precipitate a spinal headache.

The nurse is preparing to assess a patient with a head injury. Which data should the nurse include in this routine neurological nursing assessment? a. Vital signs, lung sounds, and pedal pulses b. Glasgow Coma Scale, pupil response, and vital signs c. Range of motion, deep tendon reflexes, and capillary refill d. Romberg test, Babinski reflex, and cranial nerve assessment

ANS: B Assessment of neurological status minimally includes Glasgow Coma Scale score, pupil responses, muscle strength, and vital signs. A. C. Additional assessment of body systems are important but are not part of a neurological assessment. D. Romberg, Babinski, and cranial nerve assessment is more advanced and not routine.

The nurse is caring for a patient admitted to the emergency department with massive trauma to the right frontal lobe of the brain. Which data should the nurse collect related to the location of the injury? a. Presence of intact smell b. Presence of intact pupillary reflex c. Ability to remember the name of the current president d. Ability to use extraocular muscles (EOMs) of the eyes

ANS: C The cerebral cortex, and therefore the frontal lobe, is involved in thinking, learning, and memory. A. B. D. Olfactory sense, pupils, and EOMs are controlled by cranial nerves.

A patient with a spinal cord injury is unable to move the extremities. In which area should the nurse suspect that this clients injury occurred? a. L1L4 b. C4C8 c. T8T11 d. Above C4

ANS: B Cervical cord injuries can affect all four extremities, causing paralysis and paresthesias, impaired respiration, and loss of bowel and bladder control. D. If the injury is at C3 or above, the injury is usually fatal because muscles used for breathing are paralyzed. An injury at the fourth or fifth cervical vertebrae affects breathing and may necessitate some type of ventilatory support. A. C. Thoracic and lumbar injuries affect the legs, bowel, and bladder.

A patient with Bells palsy has lost 10 pounds since being diagnosed. Why should the nurse plan interventions to address the risk for nutrition problems? a. Appetite is diminished. b. Taste and chewing are affected. c. Nutrients are not absorbed efficiently. d. The patient may have difficulty preparing foods.

ANS: B Difficulty eating may cause the patient to eat less and lose weight. A, C, and D are not affected by Bells palsy.

The nurse is preparing a patient for an electroencephalogram (EEG). What information should be given to the patient? a. Little needles will be stuck into the scalp. b. The hair must be clean and dry before the test. c. The hair at the temporal area will have to be shaved. d. The patient must withhold fluids and food for 12 hours before the test.

ANS: B Evaluation of the electrical activity of the brain is obtained through an EEG. Electrodes are attached to the scalp with an adhesive. Before the test, make sure that the patients hair is clean and dry. C. D. Shaving and fluid restriction are not necessary. A. Needles are not inserted into the scalp for an EEG.

A patient diagnosed with Guillain-Barr syndrome (GBS) asks how the disease developed since the patient rarely has an illness. What nursing response is the most accurate? a. No one knows what causes it. b. It may be an autoimmune reaction to a virus. c. It most often occurs as a result of a bacterial infection. d. It is usually hereditary. Does anyone in your family have it?

ANS: B GBS is believed to be caused by an autoimmune response to some type of viral infection or vaccination. A. No one does have an exact cause however this statement is not the most accurate. C. It is believed to occur after a viral infection. D. It is not hereditary.

A patient is incontinent during a seizure and sleeps for several hours afterward. What type of seizure did the patient most likely experience? a. Absence b. Tonic-clonic c. Simple partial d. Status epilepticus

ANS: B Generalized tonic-clonic seizures follow a typical progression. Aura and loss of consciousness may or may not occur. The patient is often incontinent. Patients who experience a generalized seizure may sleep deeply for 30 minutes to several hours. A. C. Absence and simple partial seizures are not generally associated with incontinence. D. A patient in status epilepticus may be incontinent but would still be having a seizure and not sleeping

The nurse is caring for a patient experiencing an acute exacerbation of multiple sclerosis (MS). Which pathophysiological change should the nurse recognize as causing the manifestations of MS? a. Myelin buildup in the central nervous system b. Demyelination and destruction of nerve fibers c. Gamma aminobutyric acid (GABA) deficiency d. Reduced acetylcholine receptors with impaired nerve impulse transmission

ANS: B In multiple sclerosis, the myelin sheath begins to break down or degenerates as a result of the activation of the bodys immune system. The nerve becomes inflamed and edematous. Nerve impulses to the muscles slow down. As the disease progresses, sclerosis or scar tissue damages the nerve. A. Myelin does not build up. C. GABA is an inhibitory neurotransmitter. D. Acetylcholine receptors are damaged in myasthenia gravis.

The nurse is helping a patient with trigeminal neuralgia with bathing. Which action should the nurse take when washing the patients face? a. Use hot water and antibacterial soap. b. Use lukewarm water and cotton balls. c. Use cold water to reduce inflammation. d. Use a washcloth to stimulate circulation.

ANS: B Lukewarm water and soft cotton may be less likely to exacerbate pain. A. C. D. A rough washcloth or temperature extremes can cause pain.

A patient is scheduled for a thymectomy. For which peripheral nervous system disorder should the nurse plan care for this patient? a. Multiple sclerosis (MS) b. Myasthenia gravis (MG) c. Guillain-Barr syndrome (GBS) d. Amyotrophic lateral sclerosis (ALS

ANS: B No cure has been found for MG. Treatment is aimed at control of symptoms. Removal of the thymus gland (thymectomy) can decrease production of anticholinesterase (ACh) receptor antibodies and decrease symptoms in most patients. A. C. D. A thymectomy is not indicated in the treatment of MS, GBS, or ALS.

A patient with a severe headache due to viral meningitis requests an opioid analgesic. What explanation about opioids should the nurse provide? a. Opioid analgesics increase intracranial pressure. b. Opioid analgesics are used as a last resort for headaches. c. Opioid analgesics are contraindicated in patients with meningitis. d. Acetaminophen (Tylenol) is more effective in treating meningitis-related headaches.

ANS: B Opioids are habit forming and are used only as a last resort for headaches. A. C. They do not increase intracranial pressure, and they are not contraindicated for other reasons. D. Tylenol is much less effective than an opioid.

The nurse is preparing a patient with myasthenia gravis to undergo plasmapheresis. Which laboratory tests should the nurse verify and place on the medical record before the procedure? a. Urine analysis, urine protein, BUN, and creatinine b. Complete blood count, platelets, and clotting studies c. Creatinine phosphokinase, blood type, and electrolytes d. Electrolytes, blood urea nitrogen (BUN), creatinine, and albumin

ANS: B Plasmapheresis is used to remove the patients plasma and replace it with fresh plasma. Complete blood cell count, platelet count, and clotting studies are assessed prior to the procedure. A. B. D. Laboratory tests such as urinalysis, urine protein, BUN, creatinine, blood type, electrolytes, and albumin are not necessary before having a plasmapheresis.

A student under a great deal of stress develops a severe tension headache and goes to the school clinic. What strategy should the nurse teach the student for dealing with the onset of headaches in the future? a. Aerobic exercise b. Relaxation exercises c. Use of vitamin C and zinc d. Use of distraction techniques

ANS: B Relaxation exercises help treat and may help stop a headache before it becomes severe. D. Distraction is good for some types of pain but less so for headaches. C. Vitamin C and zinc are good for wound healing. A. Aerobic exercise may promote general health and help the student to deal with stress but will not help once the headache has begun.

A patient arriving in the emergency department with a bullet wound to the left frontal lobe is comatose. What should the nurse make a priority for this patient? a. Evaluate fluid balance. b. Maintain an open airway. c. Maintain body temperature. d. Evaluate neurological status

ANS: B Remember the ABCs: airway is always the first priority. A. C. D. Fluid balance, body temperature, and neurological status are important however are not helpful if the patient does not have a patent airway.

The nurse is caring for a patient with an exacerbation of multiple sclerosis. Which medication should the nurse anticipate administering to this patient? a. Thyrotropin b. Pyridostigmine (Mestinon) c. Diphenhydramine (Benadryl) d. Adrenocorticotropic hormone (ACTH)

ANS: B Steroids such as ACTH, prednisone, and other corticotropic medications are given to decrease inflammation and edema of the neuron, which may relieve some symptoms. C. Benadryl is an antihistamine. B. Pyridostigmine (Mestinon) is used in myasthenia gravis. A. Thyrotropin stimulates release of thyroid hormone.

A nursing home resident with Alzheimers disease appears extremely distressed after breakfast. On which understanding should the nurse base interventions for this patient? a. The patient needs an increase in antipsychotic medications. b. The patient could quickly become more anxious and dysfunctional. c. The patient would benefit from external stimuli and diversionary activities. d. This is part of the sundowning syndrome associated with Alzheimers disease.

ANS: B Stress may increase dysfunctional behaviors. D. Sundowning is confusion that occurs at sundown, not in the morning; it has not been validated. A. Increasing medications is inappropriate if simple environment alterations can keep the patient calm. C. Stimuli make dysfunction worse, not better.

A patient with amyotrophic lateral sclerosis has difficulty swallowing and copious pulmonary secretions. What equipment should the nurse ensure is at the bedside at all times? a. Tissues b. Suction c. Oxygen d. Tongue blade

ANS: B Suction is necessary in case the patient aspirates because of difficulty swallowing. A. C. Oxygen and tissues may be helpful but will not clear the airway. D. A tongue blade is not necessary.

A 17-year-old patient with a new onset of seizures is diagnosed with epilepsy. What should the nurse include in the patient teaching? a. Aspirin can inhibit the action of anticonvulsants. b. Sudden withdrawal of anticonvulsants can lead to status epilepticus. c. Anticonvulsants must be taken frequently during the day to prevent seizures. d. When the seizures have been controlled, the medications can be discontinued.

ANS: B Sudden discontinuance of a medication can result in status epilepticus. A. Aspirin does not interfere with anticonvulsantseach drug has different interactions that should be checked and communicated to the patient. C. The schedule is also dependent on the drugsome may be needed only once or twice a day. D. Medications for epilepsy will most likely be needed lifelong.

The nurse administers an analgesic to a patient with a headache. How should the nurse assess the patients response to the medication? a. Observe the patients behavior. b. Ask the patient to describe the pain. c. Monitor the patients blood pressure and pulse. d. Have the patient rate the pain on a scale of 0 to 10.

ANS: D A pain rating scale is the most objective measure of the patients pain. A, B, and C may also be helpful but are not the primary data needed. Blood pressure and pulse may increase with acute pain but not with chronic pain.

The nurse notes that a patient is not able to voluntary move the right arm. Which part of the brain should the nurse suspect is affected in this patient? a. Cerebellum b. Frontal lobe c. Parietal lobe d. Hypothalamus

ANS: B The frontal lobes contain the motor areas that generate the impulses that bring about voluntary movement. Each motor area controls movement on the opposite side of the body. C. The parietal lobes contain the general sensory areas for the cutaneous senses, conscious muscle sense (proprioception), and taste (gustation). D. The hypothalamus has varied functions. A. The functions of the cerebellum are concerned with the involuntary aspects of voluntary movement: coordination, the appropriate direction and endpoint of movements, and the maintenance of posture and balance or equilibrium.

The nurse is caring for a patient diagnosed with a cerebral tumor. For which function should the nurse expect to assess an abnormality? a. Reflex movement b. Movement and speech c. Coordination and posture d. Heart rate and respiratory rate

ANS: B The frontal lobes of the cerebrum contain the motor areas that generate the impulses that bring about voluntary movement. Each motor area controls movement on the opposite side of the body. Also in the frontal lobe, usually only the left lobe is Brocas motor speech area, which controls the movements involved in speaking. D. The medulla controls heart and respiratory rates. C. The cerebellum controls coordination and posture. A. Reflexes are controlled at the spinal cord level.

The nurse notes that a patient has a history of falling. Which part of the brain should the nurse question as being affected in this patient? a. Medulla b. Cerebellum c. Frontal lobes d. Hypothalamus

ANS: B The functions of the cerebellum are concerned with the involuntary aspects of voluntary movement: coordination, the appropriate direction and endpoint of movements, and the maintenance of posture and balance or equilibrium. C. The frontal lobes contain the motor areas that generate the impulses that bring about voluntary movement. Each motor area controls movement on the opposite side of the body. The frontal lobes contain the motor areas that generate the impulses that bring about voluntary movement. D. The hypothalamus has many functions. A. The medulla regulates the most vital life functions.

A mother of three young children who has a 3-year history of myasthenia gravis has had to stop helping in the childrens classrooms in the morning because of fatigue. What should the nurse say to help the patient best cope with the problem? a. You need to realize that you may not be able to do the things you used to do. b. Time your medication so that its action is peaking during the time you need the most energy. c. Getting plenty of sleep the night before you are scheduled to help will give you the stamina you need. d. If you wait to take your medication after you finish helping in the classroom, you may find that your energy level is better.

ANS: B The patient should be instructed to schedule activities at times when medication is at peak action so that muscle strength is increased. A. C. These statements may also be true but medication is needed to get through an activity. D. Taking the medication after the activity will help with strength after, not during, the activity.

The nurse is caring for a patient who is scheduled for a computed tomography (CT) scan of the brain because of new onset of headaches. Which statement by the nurse is most accurate when preparing the patient for the scan? a. You must shampoo your hair thoroughly tonight to remove oil and dirt. b. You will need to hold your head completely still during the examination. c. You may take fluids until about 8 a.m. Then we will give you a special radiopaque drink. d. We will partially shave your head tonight so that electrodes can be attached securely to your scalp.

ANS: B The patient will need to hold the head still during the test. No special preparation is required for a head CT. A. Shampoo is done before electroencephalogram (EEG). C. Restricting fluids and radiopaque drinks is done prior to gastrointestinal tests. D. Electrodes are for EEGs, but shaving is not necessary.

The nurse is caring for a patient who has impaired functioning of the left glossopharyngeal (IX) nerve and the vagus (X) nerve. What intervention should the nurse plan to maintain the patients safety while diagnostic testing is being completed? a. Insert an oral airway. b. Withhold oral fluid or foods. c. Obtain a picture board and a Magic Slate. d. Apply eye patches to keep the eyes closed.

ANS: B The patients swallowing and gag reflexes are impaired and could lead to aspiration if food or fluids are given. A. C. D. Cranial nerves (CNs) IX and X do not affect the airway, the eyes, or the ability to communicate.

While walking to the bathroom a patient begins having a generalized tonic-clonic seizure. What should the nurse do first? a. Reduce external stimuli. b. Maintain the patients airway. c. Maintain the patients privacy. d. Perform a brief neurological assessment.

ANS: B The prime objective in caring for a patient experiencing a seizure is to prevent injury. Maintain a patent airway, and if possible, turn the patient on his or her side to prevent aspiration if vomiting occurs. Do not force an airway or anything else into the patients mouth once the seizure has begun. C. D. Assessment and privacy are important, but airway always takes priority. A. Reducing stimuli will not help once the seizure has begun.

The nurse is caring for residents on an Alzheimers unit. Which assessment finding indicates that a patient is in early stages of the disease? a. Agitation b. Forgetfulness c. Combativeness d. Increased intracranial pressure (ICP)

ANS: B The signs and symptoms of Alzheimers disease are typically broken down into three stages. The early stage, stage one, lasts 2 to 4 years and is characterized by increasing forgetfulness. A. C. D. Behavior changes occur later, and increased ICP is not associated with Alzheimers.

The nurse is assisting with discharging a patient with myasthenia gravis after hospitalization for severe respiratory distress. Which patient statement indicates that the nurses discharge teaching has been effective? a. If I develop muscle cramping, I can take quinine as needed. b. I have to take my Prostigmin exactly as prescribed without skipping a dose. c. I know I should take my Prostigmin as needed, whenever I feel short of breath. d. I will take my anticholinergic medication to prevent developing respiratory distress again.

ANS: B With insufficient anticholinesterase medication, muscles can become weak. If respiratory muscles are affected, the patient can develop respiratory distress. C. Medication must be taken consistently to prevent weakness, not prn. A. D. Quinine and anticholinergic agents can exacerbate muscle weakness.

The nursing diagnosis Ineffective Airway Clearance has been identified for a patient with multiple sclerosis. Which interventions should the nurse include in this patients plan of care? (Select all that apply.) a. Measure intake and output b. Elevate the head of the bed c. Evaluate gag reflex every shift d. Monitor oxygen saturation twice a shift e. Encourage deep breathing and coughing every 2 hours

ANS: B, C, D, E Nursing interventions appropriate for the patient with Ineffective Airway Clearance include elevating the head of the bed, evaluating gag reflex every shift, monitoring oxygen saturation twice a shift, and encouraging deep breathing and coughing every 2 hours. A. Intake and output measurement would be more appropriate for the patient with a fluid volume imbalance

The nurse is caring for a patient scheduled for a lumbar puncture. Which actions should the nurse anticipate providing? (Select all that apply.) a. Position the patient prone on the bed. b. Check the puncture site for swelling or drainage. c. Ensure that the patient has given informed consent to the procedure. d. Keep the patient on bedrest with the head of the bed flat for 6 hours after the procedure. e. Limit fluid intake. f. Assess movement and sensation of lower extremities frequently for several hours after the procedure.

ANS: B, C, D, F Ensure that informed consent has been obtained prior to the procedure. A. Assist the patient into a side-lying (not prone) position with his or her back as close to the edge of the bed nearest the practitioner as possible. After the lumbar puncture is completed, instruct the patient to remain on bedrest with the head of the bed flat for 6 to 8 hours, as ordered by the physician. E. Oral intake of fluids should be increased. Keeping the head flat decreases the likelihood of leakage of cerebrospinal fluid from the puncture site, which can result in a severe headache. Increasing fluid intake promotes replacement of the fluid that was removed. Check the puncture site for swelling or drainage of cerebrospinal fluid and report any leakage to the health care provider. Assess the movement and sensation to the lower extremities frequently for the first 4 hours after the procedure. Assess the patient for headache, and if necessary, obtain an order for analgesia.

A patient is diagnosed with a benign familial tremor. Which characteristics of this tremor should the nurse expect to observe? (Select all that apply.) a. Resting tremor b. Intention tremor c. Pill-rolling tremor d. Head/voice tremor e. Relieved by beta blocker drugs

ANS: B, D, E Patients with familial tremor experience an intention tremor and head and voice tremors; symptoms may be improved with beta blockers. A. C. A resting, pill-rolling tremor is common with Parkinsons disease.

When the nurse shines a light in a patients left pupil, both of the pupils constrict. What type of response should the nurse document? a. Direct b. Abnormal c. Consensual d. Accommodation

ANS: C A consensual response means that when one pupil is exposed to direct light, the other pupil also constricts. B. This is a normal response. A. A direct response means the pupil exposed to light constricts. D. Accommodation is the process of visual focusing from far to near.

The nurse is providing care to a patient with Guillain-Barr syndrome. Which laboratory result should the nurse evaluate first? a. Electrolytes b. Blood urea nitrogen (BUN) c. Arterial blood gases (ABGs) d. Hemoglobin (Hgb) and hematocrit (Hct)

ANS: C ABGs monitor respiratory status, which is essential in case the patients respiratory muscles become affected. B. Blood urea nitrogen (BUN) monitors kidney function. D. Hgb and Hct monitor blood loss or anemia. A. Electrolytes monitor fluid and electrolyte balance.

The nurse concludes that a patients meningitis is improving. What activity did the patient perform for the nurse to come to this conclusion? a. Dorsiflex both feet. b. Sit up and drink water. c. Touch the chin to the chest. d. Maintain a side-lying position in bed.

ANS: C Ability to touch the chin to the chest indicates improvement in nuchal rigidity. A. B. D. An improvement in the patients condition is not associated with the ability to dorsiflex the feet, sit up, drink water, or maintain a sidelying position in bed.

A patient reports nearly having a motor vehicle crash and states that his heart was pounding and he was breathing heavy and fast. Currently the patients heart rate and breathing are within normal limits. Which neurotransmitter has resumed control after the patients incident? a. Serotonin b. Prostaglandin c. Acetylcholine d. Norepinephrine

ANS: C Acetylcholine mediates parasympathetic (peaceful) function. D. Norepinephrine mediates the sympathetic stress response. B. Prostaglandins are involved in pain sensation. A. Serotonin affects mood.

A patient with trigeminal neuralgia is admitted to the hospital for diagnostic testing and possible surgery. What intervention would be the most appropriate for this patient? a. Provide tissues for the patient to deal with drooling. b. Provide frequent mouth care with a firm toothbrush. c. Provide soft foods at body temperature at mealtimes. d. Provide a fan in the room to keep the room well ventilated.

ANS: C Activities such as talking, face washing, teeth brushing, shaving, and eating can cause pain in patients with trigeminal neuralgia. Soft foods at room temperature may be better tolerated than hot or cold foods. B. D. A fan or toothbrush can exacerbate pain. A. Patients with Bells palsy may drool but not patients with trigeminal neuralgia.

The nurse is caring for an individual with a head injury and notes unequal pupils. Which term should the nurse use to document this finding? a. Aphasia b. Nystagmus c. Anisocoria d. Ophthalmoplegia

ANS: C Anisocoria describes unequal pupils. A. Aphasia is difficulty speaking or communicating. B. Nystagmus is involuntary movement of the eyes. D. Some patients may be unable to move one or both eyes in a specific direction; this is called ophthalmoplegia.

The nurse is assisting as a neurosurgeon examines a patient who has a positive Babinski reflex. What assessment finding should the nurse expect to observe? a. The leg flexes when the patellar tendon is struck. b. The leg extends when the patellar tendon is struck. c. The big toe extends when the sole of the foot is stroked. d. Toes curl downward when the sole of the foot is stroked

ANS: C Babinski reflex is tested by firmly stroking the sole of the foot. If the great toe extends and the other toes fan out, neurological dysfunction should be suspected if the patient is more than 6 months old. D. Normal response is flexion of the great toe. A. B. The Babinski reflex does not assess leg flexion or extension.

A teen is experiencing a headache and dizziness after falling of a bicycle and hitting the head. The physician diagnoses a concussion. What explanation should the nurse provide to the patients mother? a. The patient may lose consciousness before beginning to recover. b. The patient has had some intracranial bleeding but should recover in time. c. The patient has had a minor head trauma and should recover spontaneously. d. The patient may need to have surgery to relieve increased intracranial pressure.

ANS: C Cerebral concussion is considered a mild brain injury. If there is a loss of consciousness, it is for 5 minutes or less. Concussion is characterized by headache, dizziness, or nausea and vomiting. The patient may complain of amnesia of events before or after the trauma. On clinical examination, there is no skull or dura injury and no abnormality detected by computed tomography (CT) or magnetic resonance imaging (MRI). A. B. D. These statements explain more serious head injuries.

A patient with a newly diagnosed brain tumor receives dexamethasone (Decadron) IV, which completely relieves the patients symptoms. What should the nurse explain to the family about the patients response to the medication? a. The brain is such a unique organ; we never really know what will happen. b. By dilating the arteries in the brain, blood flow is improved and symptoms improve. c. The Decadron works to reduce swelling in the brain caused by the tumor; we often see remarkable improvement. d. Decadron regenerates neurons in the central nervous system, so the patient should continue to get even better over the next week or so.

ANS: C Dexamethasone is a steroid that may reduce brain swelling. A. B. D. Dexamethasone is not a vasodilator, and it does not regenerate nerve tissue.

The nurse is notes that a patient recovering from a craniotomy has a pink spot with a yellow ring around it on the pillow. What should the nurse do? a. Change the patients pillowcase. b. Do a basic neurological assessment. c. Notify the charge nurse immediately. d. Change the patients cranial dressing.

ANS: C Drainage that is blood-tinged in the center with a yellowish ring around it may be cerebrospinal fluid (CSF) leakage. A suspected CSF leak should be reported to the charge nurse or physician immediately. A. B. These actions can be completed after the charge nurse has been notified. D. The dressing is changed only with a physicians order.

The nurse is preparing material about impulse transmission to help with a presentation on the neurological system. When discussing spinal nerves, the nurse will include that each spinal nerve is made up of the dorsal root and which other root? a. Medial b. Lateral c. Ventral d. Proximal

ANS: C Each spinal nerve has two roots, which are neurons entering or leaving the spinal cord. The dorsal root is made of sensory neurons that carry impulses into the spinal cord. The dorsal root ganglion is an enlargement of this root that contains the cell bodies of these sensory neurons. The ventral root is the motor root; it is made of motor neurons that carry impulses from the spinal cord to muscles or glands. (Their cell bodies are in the gray matter of the spinal cord.) When the two roots merge, the nerve thus formed is a mixed nerve. A. B. D. Medial, lateral, and proximal are not parts of a spinal nerve.

The nurse is caring for a patient with a traumatic brain injury. Which assessment finding alerts the nurse to possible diabetes insipidus? a. Headache b. Confusion c. Frequent urination d. Elevated blood glucose

ANS: C Edema or direct injury affects the posterior portion of the pituitary gland or hypothalamus. Inadequate release of antidiuretic hormone results in polyuria and, if the patient is awake, polydipsia. Fluid replacement and intravenous vasopressin are used to maintain fluid and electrolyte balance. A. B. Headache and confusion are symptoms of intracranial pressure (ICP). D. Elevated glucose is a sign of diabetes mellitus, not insipidus.

The nurse is providing eye care to a patient with Bells palsy. Which nursing action is most appropriate to help protect the affected eye? a. Keep the room lights dim at all times. b. Check for pupil response twice a day. c. Request an order for ointment and a patch. d. Apply a warm moist compress to the eye three times a day.

ANS: C Eyedrops and a patch can help keep the eye closed and protect the patients eye. D. A warm compress will not protect the eye, especially if it is only used three times a day. A. B. Dimming the lights and checking pupils will not protect the eye.

A patient with a newly diagnosed seizure disorder is being prepared for discharge. What medication should the nurse anticipate will be prescribed for the patient to prevent recurrent seizures? a. Selegiline (Eldepryl) b. Haloperidol (Haldol) c. Gabapentin (Neurontin) d. Dexamethasone (Decadron)

ANS: C Gabapentin is an anticonvulsant agent. D. Dexamethasone is a steroid. B. Haloperidol is an antipsychotic agent. A. Selegiline is used to treat Parkinsons disease.

The nurse is assessing a patient recovering from a tonic-clonic seizure. Which finding indicates a need for immediate nursing intervention? a. The patient is difficult to arouse. b. The patient has been incontinent of urine. c. The patient has frothy sputum in the pharynx and gurgling respirations. d. The patient becomes belligerent when the nurse does neurological assessments.

ANS: C Gurgling respirations and frothy sputum indicate aspiration, and a clear airway is a priority. A. B. Difficulty arousing and incontinence of urine are not uncommon findings after a tonic-clonic seizure. D. Belligerence is a concern but is not life threatening.

The physician prescribes intravenous mannitol for a patient who has a head injury and increased intracranial pressure (ICP). Which assessment finding indicates to the nurse that the patient is having a therapeutic response to the mannitol? a. Return of the gag reflex b. Increased blood glucose c. Increased urinary output d. Decreased Glasgow Coma Scale (GCS) score

ANS: C If ICP remains elevated despite drainage of cerebrospinal fluid, the next step is use of an osmotic diuretic. The most commonly used drug is intravenous mannitol (Osmitrol). Mannitol utilizes osmosis to pull fluid into the intravascular space and eliminate it via the renal system. D. The GCS score should increase, not decrease. A. B. Blood glucose and gag reflex are not affected by mannitol.

A patient who is prescribed neostigmine (Prostigmin) for newly diagnosed myasthenia gravis (MG) asks how the medication works. What should the nurse respond to the patient? a. It is a muscle relaxant that prevents the cramping in your muscles. b. It provides potassium to your muscles so that they will contract better. c. It makes more neurotransmitter available so that your muscles can contract. d. It reduces the inflammation in your nerves so that they transmit signals better.

ANS: C Medications used to treat MG include the anticholinesterase (ACh) drugs neostigmine (Prostigmin) and pyridostigmine (Mestinon). These drugs improve symptoms of MG by destroying the acetylcholinesterase that breaks down ACh. A. B. This medication does not provide potassium or relax the muscles. D. Steroids reduce inflammation.

The nurse is reviewing nursing diagnoses identified for a patient with trigeminal neuralgia. Which nursing diagnosis should the nurse identify as a priority for this patient? a. Ineffective coping b. Self-care deficit: hygiene c. Pain related to inflammation of cranial nerve V d. Imbalanced nutrition: less than body requirements

ANS: C Pain takes priority. D. If pain is controlled, then the patient will be able to eat, and nutrition will not be a problem. A. Ineffective coping can be addressed after physiological problems. B. Hygiene is important but is not as high a priority as pain and nutrition.

A patient is unable to move the extremities after experiencing a spinal cord injury. What term should the nurse use to document paralysis of all four extremities? a. Paraplegia b. Hemiparesis c. Quadriplegia d. Quadriparesis

ANS: C Paralysis of all four extremities is called quadriplegia. D. Weakness of all extremities is called quadriparesis. A. Paraplegia is paralysis of the lower extremities. B. Hemiparesis is weakness of one side.

The nurse is assisting with teaching a patient about tension headaches. Which explanation of tension headaches should the nurse provide? a. Tension headaches result from release of pain mediators in the periphery. b. Tension headaches are caused by stress, which causes cerebral vessel constriction. c. Tension headaches are a result of stress and sustained muscle contraction of the head and neck. d. Tension headaches are caused by blood sugar fluctuations that result from excessive stress.

ANS: C Persistent contraction of the scalp, facial, cervical, and upper thoracic muscles can cause tension headaches. A cycle of muscle tension, muscle tenderness, and further muscle tension is established. B. A migraine headache is believed to be caused by cerebral vasoconstriction followed by vasodilation. A. Pain mediators in the periphery are associated with peripheral pain, not headaches. D. Blood sugar fluctuations can cause headaches related to diabetes, not stress.

A patient with Parkinsons disease has difficulty tying shoes. What nursing intervention would be the most helpful? a. Tie the shoes for the patient. b. Reteach the patient to tie shoes. c. Have a family member purchase shoes with Velcro fasteners. d. Explain to the patient that as the disease progresses, there will be many things that will require assistance.

ANS: C Providing Velcro fasteners allows the patient to remain independent as long as possible. B. Re-teaching is not appropriate: The patient has not forgotten how to tie his shoes; he is unable because of motor difficulties. A. Typing the patients shoes takes away his independence. D. Explaining that the patient will need more assistance as the disease progresses does not address the current issue of not being able to tie the shoes.

The nurse is caring for a patient who has had a stroke (brain attack). The patient is unable to understand what the nurse is saying and appears frustrated. What term should the nurse use to document this finding? a. Dysphagia b. Confusion c. Receptive aphasia d. Expressive aphasia

ANS: C Receptive aphasia affects the patients ability to understand spoken language. D. Expressive aphasia is difficulty or inability to communicate verbally with others. A. Dysphagia is difficulty swallowing. B. A patient who is unable to communicate is not necessarily confused.

The nurse caring for patients with dementia. Which intervention would be least helpful when coordinating care for patients who are experiencing confusion? a. Providing finger foods b. Monitoring cognitive functioning c. Using soft restraints when the patient is left alone d. Providing structured rest periods to prevent fatigue

ANS: C Restraints can increase the risk of agitation and injury. A. B. D. Finger foods, structured rest periods, and monitoring are all helpful when caring for a patient with confusion.

A patient in the post-ictal period after a seizure remembers smelling something like dead fish prior to the seizure. Which response by the nurse is best? a. Today is Friday; the hospital always cooks fish on Fridays. b. You were probably hallucinating; I will ask for an order for an anti-hallucinatory agent. c. The smell of dead fish might be your aura; you should call for help immediately if you smell it again. d. Most people see a flash of light before a seizure; if this occurs, you should get to safety immediately.

ANS: C Some patients experience an aura or sensation that warns the patient that a seizure is about to occur. An aura may be a visual distortion, a noxious odor, or an unusual sound. Patients who experience an aura may have enough time to sit or lie down before the seizure starts, thereby minimizing the chance of injury. B. Treating the aura as a hallucination does not help the patient to get to safety. A. Commenting on a food item on the menu is not appropriate. D. Flashes of light can occur however a noxious odor can also be an aura.

The nurse is caring for a patient admitted to the emergency room after a motor vehicle crash. Which assessment is most important for the nurse to complete? a. Babinski test b. Romberg test c. Glasgow Coma Scale d. Visual analogue scale

ANS: C The Glasgow Coma Scale is an international scale used to assess level of consciousness and document findings and would be the highest priority. D. A visual analog scale is often used to measure pain level. A. B. Babinski and Romberg tests assess muscle function.

The nurse is assisting a neurologist with assessment of a patient with facial muscle weakness. When the neurologist asks the patient to identify different odors, which nerve is being tested? a. II (optic) b. X (vagus) c. I (olfactory) Test Bank - Understanding Medical-Surgical Nursing 6e (Williams and Hopper) 766 d. VIII (acoustic)

ANS: C The olfactory nerve controls the sense of smell. A. The optic nerve controls sight. D. The acoustic nerve controls hearing and equilibrium. B. The vagus nerve controls some autonomic functions.

A patient who was in an industrial accident has had a sudden increase in intracranial pressure and is being prepared for placement of an emergency subarachnoid bolt. Which action should the nurse make a priority at this time? a. Find out how the accident happened. b. Ensure the patient is bathed before surgery. c. Have the patients next of kin sign a consent form. d. Send the patients belongings home with a family member.

ANS: C The patient is unlikely to be able to sign a consent form, and it must be signed for surgery to begin. B. Bathing is not a priority. A. D. Belongings and further questioning can be taken care of after the patient is in surgery.

The nurse is explaining the neurological system to a group of nursing students. How many pairs of spinal nerves should the nurse explain are contained within the human body? a. 15 b. 25 c. 31 d. 42

ANS: C There are 31 pairs of spinal nerves, named according to their respective vertebrae: 8 cervical pairs, 12 thoracic pairs, 5 lumbar pairs, 5 sacral pairs, and 1 very small coccygeal pair. A. B. There are more than 15 or 25 pairs of spinal nerves. D. There are not 42 pairs of spinal nerves in the body.

The nurse notes that a patient with a head injury has a widening pulse pressure. Which action should the nurse take at this time? a. Give an extra dose of diuretic. b. Lay the bed flat and check pupil response. c. Raise the head of the bed and notify the registered nurse (RN). d. None; this is an expected finding after a head injury.

ANS: C Widening pulse pressure or falling blood pressure are signs of increased intracranial pressure (ICP) and should be reported promptly. B. Raising the head of the bed 30 degrees may help reduce ICP. D. Increased ICP is not unexpected, but it is not normal and must be reported. A. A diuretic would only be given with a physicians order.

A patient has been prescribed the dopamine agonist pramipexole (Mirapex) for Parkinsons disease. Which are important for the nurse to include when teaching about this medication? (Select all that apply.) a. Take it at noon each day. b. Increase fluids and fiber in your diet. c. Taking the medication with food may reduce nausea. d. You may experience sudden bouts of excessive sleepiness. e. Do not drive until the effects of this drug on you are fully known. f. Because this drug may interact with some painkillers, be sure to tell health care providers that you are taking Mirapex

ANS: C, D, E Patients may fall asleep suddenly when taking this medication. The patient should be cautioned to avoid driving until effects are known. Giving with meals may reduce nausea. B. It is unknown if this medication causes constipation. A. F. Selegiline, not pramipexole, should be given at noon and interacts with meperidine (Demerol).

A patient with multiple sclerosis has been prescribed baclofen (Lioresal) to relax muscles. What should be included in the nurses teaching about this drug? (Select all that apply.) a. Avoid crowds while on this medication. b. Take a calcium supplement while on this medication. c. Report any shortness of breath or other respiratory problems. d. Do not drive or operate machinery until the effects of the drug on you are known. e. Do not eat grapefruit, drink grapefruit juice, or consume other products containing grapefruit. f. Try to prevent constipation by adequate fluid intake, eating fiber-rich foods, and using suppositories occasionally if necessary

ANS: C, D, F Patients taking antispasmodics such as baclofen should avoid operating machinery and driving until effects are known. Measures should be provided to prevent constipation (except dantrolene). The patient should be monitored for respiratory depression. A. B. Calcium supplements are helpful with steroids, as is avoiding crowds due to infection risk. E. There is no restriction to grapefruit with this medication.

A patient is surprised to learn that cerebrospinal fluid will be removed during a lumbar puncture and asks the purpose of the fluid. What should the nurse explain to the patient? (Select all that apply.) a. Interprets sensory information b. Provides oxygen to the brain tissue c. Cushions the central nervous system d. Conducts electrical impulses between brain hemispheres e. Exchanges nutrients and wastes between the blood and neurons

ANS: C, E Cerebrospinal fluid permits the exchanges of nutrients and wastes between the blood and CNS neurons. It also acts as a cushion or shock absorber for the CNS. A. The cerebrospinal fluid does not interpret sensory information. B. Blood and not cerebrospinal fluid provides oxygen to brain tissue. D. Nerves and not cerebrospinal fluid conduct electrical impulses between the brain hemispheres.

The nurse is preparing a review of the neurological system as part of a community health presentation. Which structures should the nurse identify as being part of the diencephalon? (Select all that apply.) a. Pons b. Medulla c. Thalamus d. Brainstem e. Hypothalamus

ANS: C, E The diencephalon consists primarily of the thalamus and hypothalamus. D. The diencephalon is superior in structure to the brainstem. A. B. The medulla and pons are structures within the brainstem

The nurse is assisting with the administration of a Tensilon test. What response to the test causes the nurse to suspect that the patient has myasthenia gravis? a. Dyspnea develops b. Muscle cramps develop c. Muscles become very weak. d. Ptosis is temporarily improved

ANS: D A test for myasthenia gravis involves an intravenous injection of edrophonium (Tensilon, an anticholinesterase drug). If muscle strength improves dramatically (e.g., the patient can suddenly open the eyes wide), MG is diagnosed. A. B. C. Dyspnea, weakness, or cramping is not expected after Tensilon administration.

The nurse is caring for a patient with an exacerbation of multiple sclerosis (MS). What should the nurse include when teaching the patient about risk factors for exacerbation? a. Vegetarian diet b. Exposure to sun c. Sedentary lifestyle d. Urinary tract infection

ANS: D A variety of factors can trigger the onset of symptoms or aggravate the condition, including extreme heat and cold, fatigue, infection, and physical and emotional stress. A. B. C. Sun, sedentary lifestyle, and vegetarian diet are not known to exacerbate multiple sclerosis.

The nurse is assisting with teaching a patient who has had a transient ischemic attack (TIA). On which understanding should the nurse base teaching? a.TIAs are not serious, and the patient should have no further problems. b.A TIA is predictive that the patient will have a heart attack within 1 year. c.A TIA is a medical emergency that requires immediate surgical intervention. d.A TIA is a forewarning that the patient is at risk for a cerebrovascular accident (stroke).

ANS: D About a third of patients who experience a TIA will have a stroke in the future. A. Urgent evaluation of TIA is essential in order to decrease the risk of stroke. B. There are no data related to myocardial infarction (MI) prediction. C. It is not a surgical problem.

The nurse is assisting with teaching family members about a patients epidural bleed. Which information about an epidural bleed should guide the nurses teaching? a. It is usually venous and absorbs in time. b. It is within the brain tissue, so residual effects are likely. c. It usually causes quadriplegia, and rehabilitation will be necessary. d. It is usually arterial and may lead to death without rapid intervention.

ANS: D An epidural bleed is usually arterial in nature. Arterial bleeding can cause the hematoma to become large very quickly. The patient will die without rapid intervention. A. Subdural bleeds are more likely venous. B. A contusion is bruising of brain tissue. C. Paralysis depends on the area of central nervous system (CNS) injured.

A patient is recovering from an epidural bleed. In which part of the brain should the nurse explain to the family that this bleed occurred? a. Circle of Willis b. Spinal meninges c. Space below the dura d. Space between the dura and the skull

ANS: D An epidural hematoma is a collection of blood between the dura mater and skull, is usually arterial in nature, and is often associated with skull fracture. A subdural hematoma is typically venous in nature and accumulates between the dura and arachnoid membranes. A. B. C. An epidural bleed does not occur within the Circle of Willis, spinal meninges or the space below the dura.

A neurologist asks a patient to stick out the tongue. Which cranial nerve (CN) is being tested? a. VII (facial) b. I (olfactory) c. IV (trochlear) d. XII (hypoglossal)

ANS: D CN XII controls tongue movement. B. CN I controls sense of smell. C. CN IV is eyeball movement. A. CN VII is taste and facial muscles.

The results of a carotid Doppler study indicate that a patient has stenosis of the left carotid artery. For which diagnostic test should the nurse prepare the patient to have completed next? a.MRI b.CT scan c.Echocardiogram d.Carotid angiography

ANS: D Carotid Doppler testing uses ultrasound to detect stenosis of the carotid arteries. Carotid angiography can be done to further determine degree of blockage and help guide treatment. A. B. C. After a carotid Doppler, an MRI, CT scan, or echocardiogram are not indicated.

While collecting data the nurse learns that a patient with a neurological illness has not had a sense of smell for several decades. Which part of the central nervous system should the nurse question as being damaged in this patient? a. Brainstem b. Occipital lobe c. Hypothalamus d. Temporal lobe

ANS: D Cranial nerve I is the olfactory nerve and originates in the temporal lobe. A. C. Cranial nerves do not originate from the brainstem or hypothalamus. B. Cranial nerve II is the optic nerve and originates in the occipital lobe of the cerebrum.

A patient is scheduled for a lumbar puncture. Which action should the nurse take when preparing this patient? a. Remove all metal jewelry. b. Administer enemas until clear. c. Remove the patients dentures. d. Assist the patient into a side-lying position.

ANS: D For a lumbar puncture, the nurse assists the patient into a side-lying position with his or her back as close to the edge of the bed nearest the practitioner as possible. Depending on the patients condition, the nurse may need to help the patient flex his or her knees up to the chest. B. Enemas are for gastrointestinal (GI) tests. A. Metal must be removed for magnetic resonance imaging (MRI). D. Dentures are removed for surgeries or endoscopic procedures.

The nurse suspects that a patient has vision changes caused by a stroke. What did the nurse assess to make this determination? a.Patient asks that all items be placed on the right side of the bed. b.Patient turns head away when blood is being drawn from an arm. c.Patient looks down at the floor when sitting on the side of the bed. d.Patient does not follow with the eyes as the nurse walks around the room.

ANS: D If the patients eyes do not follow the nurse when moving around the room, there is a good chance that the patient has a deficit in that visual field. A. B. C. Placing items on the right side of the bed, turning the ahead away while blood is being drawn, and looking down at the floor when sitting on the side of the bed do not indicate vision changes caused by a stroke.

The nurse is caring for a patient brought to the emergency department after an automobile accident. The patient is fully conscious. For what early signs of increased intracranial pressure (ICP) should the nurse be alert? a. Bradycardia b. Hypothermia c. Pinpoint pupils d. Decreased level of consciousness

ANS: D Initial symptoms of increased ICP include restlessness, irritability, and decreased level of consciousness, because cerebral cortex function is impaired. If not intubated, the patient may hyperventilate, causing vasoconstriction as the body attempts to compensate. As the pressure increases, the oculomotor nerve may be compressed on the side of the impairment. C. Compression of the outermost fibers of the oculomotor nerve results in diminished reactivity and dilation of the pupil. As the fibers become increasingly compressed, the pupil stops reacting to light. If the compression continues, and the brain tissue exerts pressure on the opposite side of the brain from the injury, both pupils become fixed and dilated. B. Hypothermia is not a sign of IICP. A. Vital sign changes are a late indication of increasing ICP.

A patient newly diagnosed with Parkinsons disease is prescribed carbidopa/levodopa (Sinemet). Which patient statement indicates teaching about the medication has been effective? a. The medication causes urinary retention and a dry mouth. b. Sinemet reduces inflammation in the central nervous system. c. I should take this medication when my hand tremors bother me. d. This medication converts to dopamine in the brain so my symptoms should improve.

ANS: D Levodopa/carbidopa (Sinemet) converts to dopamine in the brain. Carbidopa prevents peripheral breakdown of levodopa, so more is available in the central nervous system (CNS). A/ Urine retention and dry mouth are associated with anticholinergic medications such as Trihexyphenidyl (Artane). C. Sinemet should be taken consistently, not on a prn schedule. B Sinemet does not affect inflammation.

The nurse is caring for a patient who is scheduled for a magnetic resonance imaging (MRI) scan. What explanation should be provided to the patient and family? a. A scan of the brain will be done after injection of a radioisotope. b. An MRI uses electrodes placed on the scalp to measure activity of the brain. c. An MRI measures muscle contraction after stimulation by tiny needle electrodes. d. An MRI is a noninvasive test that uses magnetic energy to visualize internal parts.

ANS: D MRI uses magnetic energy to visualize soft tissues. C describes an electromyogram (EMG). B describes an electroencephalogram (EEG). A is a brain scan.

A patient has been prescribed pravastatin (Pravachol) to reduce cholesterol level after having a transient ischemic attack (TIA). What possible side effect should the nurse include when teaching the patient about this drug? a.Diarrhea b.Purple toe c.Confusion d.Muscle aches

ANS: D Muscle pain or aches can signal a serious side effect (rhabdomyolysis) and should be reported. A. B. C. Diarrhea, purple toe, and confusion are not side effects of statins.

The nurse is reviewing the results of a patients diagnostic tests which show changes in nerve insulation. What structure should the nurse explain to the patient that electrically insulates neurons? a. Astrocytes b. Gray matter c. Interneurons d. Myelin sheath

ANS: D Myelin is a phospholipid sheath that electrically insulates neurons from one another. B. C. The gray matter is where the cell bodies of motor neurons and interneurons are located. A. Astrocytes are part of the neuroglia.

A patient is diagnosed with increased intracranial pressure. What pressure measurement should the nurse expect to be associated with this diagnosis? a. 3 b. 5 c. 8 d. 17

ANS: D Normal ICP is 0 to 15 mm Hg. This pressure fluctuates with normal physiological changes, such as arterial pulsations, changes in position, and increases in intrathoracic pressure. A. B. C. These are considered normal intracranial pressure measurements.

A client with a subarachnoid bleed refuses to use a bedpan and becomes angry when denied permission to walk to the bathroom. While waiting to hear from the health care provider (HCP), which action should the nurse take? a.Help the patient to get up on a bedside commode b.Wait for the neurosurgeon to call back with orders c.Page security to restrain the patient from harming the nurse d.Administer an as-needed dose of a sedative that is ordered

ANS: D Patients with subarachnoid hemorrhage are at risk for rebleeding. A. Straining to have a bowel movement and agitation both increase the risk of rebleeding. B. The patient may need to be sedated until the physician can be contacted. C. Bringing in security will be upsetting to the patient and can also increase the risk of raising the BP and bleeding.

A patient opens the eyes to painful stimuli, makes incomprehensible sounds, and withdraws from pain. What should the nurse calculate this patients Glasgow Coma Scale score to be? a. 2 b. 4 c. 6 d. 8

ANS: D Responds to painful stimuli 2 + makes incomprehensible sounds 2 + withdraws from pain 4 = 8. A. B. C. These are inaccurate calculations of the Glasgow Coma Scale score based upon the patients findings.

A patient learns that abdominal pain is originating from the liver. The nurse should explain to the patient that the impulses from receptors in the internal organs to the central nervous system are transmitted from which type of neurons? a. Interneurons b. Efferent neurons c. Somatic sensory neurons d. Visceral sensory neurons

ANS: D Sensory neurons from receptors in internal organs are called visceral sensory neurons. C. Sensory neurons from receptors in the skin, skeletal muscles, and joints are called somatic. B. Motor (efferent) neurons transmit impulses from the central nervous system to effectorsthat is, muscles and glands. A. Interneurons are found entirely within the central nervous system.

The nurse is caring for a patient who has had Parkinsons disease for 15 years. What symptoms should the nurse anticipate when assisting with a routine assessment? a. Cough, fever, and impaired airway clearance b. Intention tremor, flaccid muscles, and tachykinesia c. Hemiparesis, tremor of the head, and blurred vision d. Slow shuffling gait, difficulty swallowing, and pill-rolling tremor

ANS: D Slow shuffling gait, difficulty swallowing, and pill-rolling tremor are typical of Parkinsons disease. B. Intention tremor is more common with a familial tremor. C. Hemiparesis is most common with stroke or brain injury. A. Cough and fever are signs of respiratory illness.

The nurse is observing a patient to determine if seizure activity is status epilepticus. For what length of time should seizure activity occur for this diagnosis to be appropriate for the patient? a. 1 minute b. 5 minutes c. 20 minutes d. 30 minutes

ANS: D Status epilepticus is characterized by at least 30 minutes of repetitive seizure activity without a return to consciousness. This is a medical emergency and requires prompt intervention to prevent irreversible neurological damage. A. B. C. Seizure activity must occur for longer than 1, 5 or 20 minutes before being identified as status epilepticus.

The nurse is explaining the transmission of nerve impulses to a patient with a spinal cord injury. What should the nurse explain as the structure that carries nerve impulses at synapses? a. Cell membrane b. Depolarizations c. Schwanns cells d. Neurotransmitters

ANS: D The end of the axon is called the synaptic end bulb and contains a chemical neurotransmitter that is released into the synapse by the arrival of the electrical impulse. The neurotransmitter diffuses across the synapse and combines with specific receptor sites on the postsynaptic membrane. At excitatory synapses, the neurotransmitter makes the postsynaptic membrane more permeable to sodium ions, which rush into the cell, initiating an electrical impulse on the membrane of the postsynaptic neuron. A. B. C. Schwann cells, depolarization, and cell membranes are part of the neuron and its function, not the synapse.

A patient with a cerebral injury is experiencing increased intracranial pressure (ICP). Which intervention should the nurse use to help prevent further increasing intracranial pressure? a. Avoid touching the patient as much as possible. b. Provide stimulation such as radio and television for 12 hours each day. c. Provide as much nursing care at one time as possible to allow the patient to rest. d. Space nursing care at intervals so that necessary care is distributed evenly throughout a shift.

ANS: D The nurse should space care activities to provide rest between each disturbance. C. Clustering care may raise ICP. B. Stimulation can also raise ICP. A. Avoiding touch is not necessary.

The nurse is participating in the preparation of a seminar on the neurologic system for a community health fair. Which part of the system is the nurse referring when the statement rest and digest is included? a. Left hemisphere of the cerebral cortex b. Right hemisphere of the cerebral cortex c. Sympathetic division of the autonomic nervous system d. Parasympathetic division of the autonomic nervous system

ANS: D The parasympathetic division dominates during relaxed, non-stressful situations to promote normal functioning of several organ systems. Digestion proceeds normally, with increased secretions and peristalsis; defecation and urination may occur. C. The sympathetic division is dominant in stressful situations such as fear, anger, anxiety, excitement, and exercise. A. B. The left and right hemispheres of the cerebral cortex would not specifically participate in the resting or digestion processes.

A patient recovering from surgery to remove a brain tumor is found jerking rhythmically in the bed and unresponsive to verbal stimuli. What should the nurse do first? a. Call the physician. b. Find another nurse to assist. c. Hold the patient firmly to keep the patient from injuring someone. d. Protect the patient from injury and observe the sequence of events.

ANS: D The prime objective in caring for a patient experiencing a seizure is to prevent injury to the patient. A. B. Once the patients safety has been assured, another nurse can be called to assist and the physician can be notified if necessary. C. Holding the patient increases the risk of injury.

The spouse of a patient with a C7 spinal cord injury provides all care for the patient in addition to caring for three children. Which outcome criteria should the nurse identify as relevant for a nursing diagnosis of Caregiver Role Strain for this patients plan of care? a. Caregiver maintains patients health. b. Caregiver accepts constructive criticism. c. Caregiver accepts responsibility for own actions. d. Caregiver identifies resources available to assist with care.

ANS: D The spouse needs help, and the nurse can help identify resources to assist her. A. B. C. These outcome statements do not address the caregiver role strain diagnosis.

A patient becomes startled when the alarm rings for a fire drill. After reassuring the patient that there is no danger, an assessment is completed. Which finding may be related to a sympathetic response? a. Wheezing b. Confusion c. Incontinence d. Diminished bowel sounds

ANS: D The sympathetic division is dominant in stressful situations such as fear, anger, anxiety, and exercise, and the responses it brings about involve preparedness for physical activity, whether or not it is actually needed. The heart rate increases, vasodilation in skeletal muscles supplies them with more oxygen, the bronchioles dilate to take in more air and the liver changes glycogen to glucose to provide energy. Relatively less important activities such as digestion are slowed, and vasoconstriction in the skin and viscera permits greater blood flow to more vital organs such as the brain, heart, and muscles. The urethral sphincter contracts to prevent urination. A. B. C. Wheezing, confusion, and incontinence are not associated with a sympathetic response.

A patient is experiencing bilateral hemiparesis, dysphasia, visual changes, and altered level of consciousness, ataxia, and dysphagia. Which artery was most likely affected in this patients stroke? a.Carotid b.Middle cerebral c.Posterior cerebral d.Vertebrobasilar/cerebellar

ANS: D These are symptoms of vertebrobasilar/cerebellar occlusion. A. B. C. Carotid and middle or posterior cerebral occlusions are not associated with ataxia or dysphagia.

A patient with Bells palsy is experiencing symptoms. Which symptom should the nurse address first? a. Changes in taste b. Speech difficulty c. Drooping of one side of the face d. Inability to close the affected eye

ANS: D While all the problems will be disturbing to the patient, inability to close the eye could cause eye damage. A. B. C. The other problems do not usually cause permanent damage, and preventive measures will not be as helpful.

A patient comes into the emergency department with symptoms of a stroke. Which medication should the nurse expect may be given to the patient if diagnostic testing confirms an ischemic stroke? a.Heparin b.Clopidogrel (Plavix) c.Warfarin (Coumadin) d.Tissue-type plasminogen activator (tPA)

ANS: D tPA is a thrombolytic agent that can break down the thrombus causing the occlusion, which can potentially prevent or completely reverse the symptoms of an ischemic stroke. A. B. C. Heparin, warfarin, and clopidogrel can help prevent clots but are not effective in breaking up an existing clot.

The nurse is caring for a patient with amyotrophic lateral sclerosis (ALS). Which assessment findings should the nurse anticipate? (Select all that apply.) a. Hemiparesis b. Bradykinesia c. Pill-rolling tremor d. Ascending paralysis e. Progressive weakness f. Decreased coordination of extremities

ANS: E, F Primary symptoms of ALS include progressive muscle weakness and decreased coordination of arms, legs, and trunk. Atrophy of muscles and twitching (fasciculations) also occur. B. C. Pill-rolling tremor and bradykinesia are symptoms of Parkinsons disease. D. Ascending symptoms occur in Guillain-Barr syndrome. A. Paralysis on one side of the body occurs in strokes.

A patient with symptoms of impending stroke is scheduled to have a cerebral angiogram. Which statement should the nurse include when assisting with patient teaching? a.This test is designed to detect vascular lesions in the brain. b.The angiogram is done to help identify swelling in the brain. c.We need to do this to evaluate electrical function of the brain. d.This test is done to examine cerebrospinal fluid for signs of bleeding.

ANS:A A cerebral angiogram may be completed to determine the patency of cerebral vessels and the status of any collateral circulation. D. A lumbar puncture is done to examine cerebrospinal fluid (CSF). B. Edema may be identified by radiography. C. An electroencephalogram (EEG) shows electrical function.

The nurse is planning care for a patient with an intracerebral hemorrhage. What should be identified as a goal for this patient? a.Maintain blood pressure below 120/80 mm Hg b.Resume activities of daily living as soon as possible c.Expect to experience transient numbness and tingling d.Receive thrombolytic medication therapy within an hour

ANS:A An intracerebral hemorrhage is usually caused by uncontrolled hypertension. Maintaining blood pressure below 120/80 mm Hg should be the goal for these patients. B. These patient cannot resume activities of daily living until the bleeding is controlled within the brain. C. Transient numbness and tingling could indicate additional brain damage from bleeding. D. Thrombolytic therapy is not indicated for an intracerebral hemorrhage.

A patient is prescribed an antiplatelet agent to prevent strokes. Which agent was this patient most likely prescribed? a.Aspirin b.Warfarin (Coumadin) c.Acetaminophen (Tylenol) d.Tissue-type plasminogen activator (tPA)

ANS:A Aspirin is a platelet aggregation inhibitor. C. Tylenol is an analgesic but does not affect platelet function. B. Warfarin is an anticoagulant. D. tPA is a thrombolytic agent.

The nurse is assisting with a community education program related to cerebral vascular accidents. What should be included in a list of symptoms that need immediate medical attention? (Select all that apply.) a.Sudden trouble seeing in one or both eyes b.Sudden severe headache with no known cause c.Sudden confusion, trouble speaking, or understanding d.Sudden loss of hearing, ringing in the ears, or stabbing ear pain e.Sudden trouble walking, dizziness, or loss of balance or coordination f. Sudden numbness or weakness of face, arm, or leg, especially on one side of the body

ANS:A, B, C, E, F The five signs or symptoms recognized by the American Heart Association/American Stroke Association include sudden numbness or weakness of face, arm, or leg, especially on one side of the body; sudden confusion, trouble speaking, or understanding; sudden trouble seeing in one or both eyes; sudden trouble walking, dizziness, or loss of balance or coordination; and sudden severe headache with no known cause. D. Sudden loss of hearing, ringing in the ears or ear pain are not manifestations of a stroke.

The LPN has been asked to help a patient eat who has impaired swallowing due to a stroke. What should be included in the plan of care? (Select all that apply.) a.Have suction equipment available. b.Stay with the patient during meals. c.Encourage the patient to eat slowly. d.Offer the patient a straw for liquids. e.Instruct the patient to try to chew on both sides of the mouth. f. Place the patient in high Fowlers position or in a chair for meals.

ANS:A, B, C, F High Fowlers position can help reduce aspiration risk. The nurse should stay with the patient and be prepared to use suction in case aspiration does occur. D. Straws and thin liquids increase risk of aspiration. E. The patient should chew on the unaffected side; if he or she chews on the affected side, it is difficult to sense the food, and pocketing may occur. Patients should be encouraged to eat slowly and chew his or her food thoroughly.

The nurse is planning information about stroke frequency as part of a community health education program. Which demographic groups should the nurse include that are at higher than average risk for stroke? (Select all that apply.) a.Pregnant women b.Asian Americans c.American Indians d.African Americans e.Men and women 75 years old or older f. Individuals who have had a transient ischemic attack (TIA)

ANS:A, C, D, E, F Some population groups, such as African Americans, American Indians, Alaskan natives, and Mexican Americans, have a higher than average risk. Recent studies indicate that the risk of stroke may be higher in women during pregnancy and the 6 weeks following childbirth. Patients who have had a TIA have an increased risk of having a stroke; about 24% to 29% of patients who experience a TIA will have a stroke within 5 years. Strokes are most common in people over the age of 75. B. Asian Americans are not as high risk.

The nurse is assisting with a community education program about stroke prevention. Which are nonmodifiable risk factors for stroke that the nurse should include? (Select all that apply.) a.Gender b.Obesity c.Diabetes d.Heredity e.Smoking f. Elevated blood lipids

ANS:A, D Gender and heredity are not modifiable. B. C. E. F. The patient can control diabetes, heart disease, diet, exercise, lipids, and smoking to some degree.

A patient is admitted to the hospital with a severe headache and photophobia. A lumbar puncture confirms a bleeding aneurysm. What nursing interventions should the nurse anticipate assisting with to prevent increased intracranial pressure (ICP) during the acute phase of illness? a.Morphine, dark glasses, and expectorants b.Quiet room, head of bed up, and stool softeners c.Coughing and deep breathing exercises and tranquilizers d.Range of motion exercises, bedside commode, and suctioning as needed

ANS:B A quiet room with minimal stressors, elevated head, and stool softeners can help reduce ICP. A. C. Morphine and tranquilizers are not usually recommended because they can make neurological assessment difficult. A. C. Expectorants can promote coughing, which can raise ICP. C. D. Exercises, moving, and suctioning can also raise ICP.

A patient began experiencing manifestations of a stroke at 0800 hours. By which time should thrombolytic medications be provided to reverse stroke symptoms? a.0900 hours b.1250 hours c.1400 hours d.1660 hours

ANS:B If a patient experiencing ischemic stroke symptoms receives treatment within 4.5 hours of symptom onset, medication can be provided to resolve the deficits. A. A patient needs to be treated within 4.5 hours and not 1 hour. C. D. This is too long to wait to provide medication to treat the symptoms of a stroke.

A patient tells the nurse that at times it seems like the mouth muscles do not want to work and the patients speech is slurred. What should the nurse realize that the patient is describing? a.Diplopia b.Dysarthria c.Dysphagia d.Dysrhythmia

ANS:B Slurred or indistinct speech because of a motor problem or lack of coordination is referred to as dysarthria. A. Diplopia is double vision. C. Dysphasia refers to difficulty swallowing. D. Dysrhythmia is an irregular heartbeat.

A patient with a temporary loss of motor function is diagnosed with a transient ischemic attack (TIA). What should the nurse include when assisting in the teaching about this health problem? a.You had a small hemorrhage in your brain. b.Your brain was temporarily deprived of oxygen. c.The neurons in your brain are tangled, so messages get mixed up. d.You have a vessel that is occluded, blocking the blood supply to your brain.

ANS:B TIA is a temporary impairment of the cerebral circulation causing neurological impairment that lasts less than 24 hours. A. A hemorrhage would cause a hemorrhagic stroke. D. A fully occluded vessel causes an ischemic stroke. C. Tangled messages refer to Alzheimers disease.

The nurse is providing care for a patient with expressive aphasia. What should the nurse expect to find in the patients plan of care? (Select all that apply.) a.Speak loudly. b.Use a picture board. c.Obtain an interpreter. d.Provide pencil and paper. e.Speak slowly and clearly. f. Gesture or pantomime the message.

ANS:B, D For expressive aphasia, pencil and paper or a picture board can help with communication. A. Speaking loudly is not helpful unless the patient has a hearing deficit also. E. F. Speaking slowly and pantomiming may be helpful for receptive aphasia, not expressive. C. Interpreters are used for language barriers, not for aphasia.

The nurse is documenting care provided to a patient with left-sided flaccidity caused by a stroke. Which term should the nurse use to document this patients motor status? a.Ipsilateral paraplegia b.Ipsilateral hemiparesis c.Contralateral hemiplegia d.Contralateral quadriparesis

ANS:C A patient with a stroke has symptoms on the opposite side of the stroke, which is called contralateral. Onesided flaccidity is called hemiplegia. A. Ipsilateral means the same side. Para refers to the lower extremities. D. Quad refers to all four extremities; B. Hemiparesis is another term for hemiplegia.

A patient enters the emergency department with right-sided weakness and vision changes. What assessment finding should be communicated to the registered nurse (RN) or HCP immediately? a.Blood glucose 150 mg/dL b.Blood pressure 148/92 mm Hg c.Onset of symptoms occurred 90 minutes ago d.History of transient ischemic attack (TIA) 3 months ago

ANS:C All the data are significant. However, the onset of symptoms is within the time frame for the patient to receive a thrombolytic. If the nurse acts quickly, the patients stroke may be able to be reversed.

A patient with a cerebrovascular accident (stroke) has left-sided flaccidity and is unable to speak but seems to understand everything the nurse says. Which term should the nurse use to document the patients communication impairment? a.Sensory aphasia b.Motor dysphagia c.Expressive aphasia d.Receptive dysphagia

ANS:C Aphasia may be expressive, in which the patient knows what he or she wants to say but cannot speak or make sense. D. Receptive aphasia is an inability to understand spoken or written words. The patient experiencing receptive aphasia is unable to understand language. B. Dysphagia refers to difficulty swallowing. A. Sensory aphasia is not a type of communication impairment.

A patient is diagnosed with a stroke that occurred at 12 noon the previous day. When should the nurse plan to begin bedside physical therapy with this patient? a.After 5 days b.Within 2 to 3 days c.By 12 noon on the current day d.At least one week after the occurrence

ANS:C Patients should be mobilized within 24 hours if possible to prevent complications of immobility. Physical and occupational therapy are provided to maximize functioning and to progress the patient toward a return to baseline functioning. A. B. D. Waiting to begin physical therapy could reduce the patients success with physical rehabilitation.

The nurse is assisting in preparing a patient for transfer to a rehabilitation facility after a stroke. What should the nurse explain as the goal for rehabilitation? a.To monitor neurological status b.To cure any effects of the stroke c.To maximize remaining abilities d.To determine the extent of neurological deficits

ANS:C Rehabilitation can help the patient maximize remaining abilities. A. D. At this point, the patients neurological status should be stable, and all the diagnostic work has been completed. B. Cure is not realistic.

The nurse is caring for a hospitalized patient who has had a stroke and is waiting to be transferred to a rehabilitation facility. What nursing action can best maximize the patients rehabilitation potential while awaiting the transfer? a.Teach the patient what to expect at the rehabilitation facility. b.Keep the patient on bedrest to conserve energy for rehabilitation. c.Call the physical therapist for bedside rehabilitation until the transfer. d.Turn the patient every 2 hours to prevent pressure ulcers and contractures.

ANS:C Rehabilitation should begin as soon as the patient is stable. Waiting until the patient is at the rehabilitation facility to begin therapy wastes valuable time. A. Teaching the patient what to expect at the rehabilitation facility will not maximize the patients rehabilitation potential. B. Keeping the patient on bedrest could cause further mobility issues. D. Turning the patient every 2 hours to prevent ulcer formation and contractures will not necessarily maximize the patients rehabilitation potential.

The patient is diagnosed with a cerebral vascular accident that has the slowest rate of recovery and the highest probability of causing extensive neurological deficits. For which type of stroke should the nurse plan care for this patient? a.Thrombotic stroke b.Cerebral aneurysm c.Subarachnoid hemorrhage (SAH) d.Reversible ischemic neurological deficit (RIND)

ANS:C SAH is caused by rupture of blood vessels on the surface of the brain. This type of infarct has the slowest rate of recovery and the highest probability of leaving the patient with extensive neurological deficits. B. Aneurysms are often asymptomatic if they do not bleed. D. RIND is reversible. A. A thrombotic stroke does not have the slowest rate of recovery.

The nurse is planning care for a client with right-sided weakness and aphasia from a transient ischemic attack (TIA). Which area of the brain should the nurse realize was affected in this client? a.Medulla b.Occipital lobe c.Left hemisphere d.Right hemisphere

ANS:C Symptom onset is sudden and generally involves one side of the bodythe side of the body opposite to the damaged area. A. B. The manifestations of right-sided weakness and aphasia would not be present if the TIA occurred in the medulla or occipital lobe. D. The client would have left-sided manifestations if the TIA occurred in the right hemisphere.

The nurse is reviewing teaching provided to a patient with transient ischemic attack (TIA). Which statement indicates that further teaching is required? a.The risk factors and symptoms of a TIA are just like those of a stroke. b.I need to stop smoking to help lower my chances of this happening again. c.My risk for Alzheimers disease is increased now, so Ill have to stop driving. d.I recognize how important it is to take my anti-hypertension medications regularly.

ANS:C There is no association between TIA and the development of Alzheimers disease. A. The risk factors, causes, and symptoms of a TIA are identical to a cerebrovascular accident (CVA). Patients who have had a TIA have an increased risk of having a stroke. Treatment, therefore, is mostly focused on minimizing the patients risk factors for a stroke. B. D. Modifiable risk factors are those risks that can be changed by treatment, such as treating high blood pressure, or by lifestyle modification, such as stopping smoking.

The nurse is involved in a blood pressure clinic in the community, and an individual with possible stroke symptoms is brought for evaluation. Which findings in the F.A.S.T. assessment indicate the need to call emergency personnel? (Select all that apply.) a.The patient sways when asked to stand still with eyes closed. b.The patient is unable to follow directions during the assessment. c.The patient is unable to repeat a stated phrase exactly as it was stated. d.The patients face shows signs of uneven symmetry when asked to smile. e.When asked to close the eyes and hold arms straight in front, one arm drifts downward.

ANS:C, D, E The acronym F.A.S.T. can help identify a stroke. Ask the person to smile: If the face droops or is uneven on one side, it is abnormal. Ask the person to close his or her eyes and hold arms out in front of him or her: If an arm cannot be raised or drifts downward, it is abnormal. Ask the person to say It is a bright and sunny day: Any difficulty understanding or speaking is abnormal. Call 911 immediately for any abnormal findings. A. These are all indicators of a possible stroke. Brain cells may be dying. B. Inability to follow instructions is a concern but is not part of the F.A.S.T. assessment.


Conjuntos de estudio relacionados

USMLE 2018/Surgery/Basic all 92 MCQs

View Set

MKTG 312 Exam Practice Questions

View Set

Operations Management Chapter One

View Set

EMS Chapter 2 Workforce safety and wellness

View Set

AP Macro: Unit 2 Measuring the Economy

View Set

English 12A - Unit Four: An American Dream

View Set